Hung Trinh Ngoc

B5:

a,c,b,d>0

CMR: \(\frac{a}{b+c}+\frac{b}{c+d}+\frac{c}{d+a}+\frac{d}{a+b}>=2\)

B6;

a,b,c>0

CMR: \(\frac{a^2}{b^2}+\frac{b^2}{c^2}+\frac{c^2}{a^2}>=\frac{a}{b}+\frac{b}{c}+\frac{c}{a}\)

Thắng Nguyễn
12 tháng 8 2017 lúc 22:16

Bài 1:

Cho a,b,c,d là các số thực dương. Tìm giá trị nhỏ nhất của biểu thức: - K2PI – TOÁN THPT | Chia sẻ Tài liệu, đề thi, hỗ trợ giải toán

Bài 2:

Áp dụng BĐT Cauchy-Schwarz ta có:

\(\left(1+1+1\right)\left(\frac{a^2}{b^2}+\frac{b^2}{c^2}+\frac{c^2}{a^2}\right)\ge\left(\frac{a}{b}+\frac{b}{c}+\frac{c}{a}\right)^2\)

Cần chứng minh \(\frac{a}{b}+\frac{b}{c}+\frac{c}{a}\ge3\)

Áp dụng BĐT AM-GM ta có:

\(\frac{a}{b}+\frac{b}{c}+\frac{c}{a}\ge3\sqrt[3]{\frac{a}{b}\cdot\frac{b}{c}\cdot\frac{c}{a}}=3\) (đúng)

Khi a=b=c

Hung Trinh Ngoc
13 tháng 8 2017 lúc 16:22

Thanks


Các câu hỏi tương tự
Dưa Dưa Tiểu
Xem chi tiết
Sakura Kinomoto
Xem chi tiết
Lê Đăng Khoa
Xem chi tiết
hoàng thị huyền trang
Xem chi tiết
Phan Mạnh Tuấn
Xem chi tiết
Nghiem Anh Tuan
Xem chi tiết
hết tên để đặt
Xem chi tiết
Nguyễn Thúy Nga
Xem chi tiết
Lê Tài Bảo Châu
Xem chi tiết